Evaluating the line integral

The question I'm working on states: Let $C$ be the curve in $\mathbb R$ consisting of line segments from $(4,1)$ to $(4,3)$ to $(1,3)$ to $(1,1)$. Let $F(x,y) = (x+y)i + (y-1)^3 * e^j$. Evaluate the line integral: $$\int_ F* \,dr$$ I've never computed the line integral with four separate points before. I'm thinking that I'll need to use Green's Theorem and use the points as boundaries for integration? However I'm not even sure how to start the process. Any help would be greatly appreciated!

23k 3 3 gold badges 43 43 silver badges 72 72 bronze badges asked Jul 11, 2015 at 19:49 user3472798 user3472798 507 1 1 gold badge 5 5 silver badges 14 14 bronze badges

$\begingroup$ Just to be clear, does the curve $C$ include the line segment from $(1,1)$ to $(4,1)$, or is $C$ an open curve? It's not going to matter too much as the problem is easily solved in either case. $\endgroup$

Commented Jul 11, 2015 at 20:19

$\begingroup$ @JimmyK4542 It doesn't specify whether it includes the line segment, so I can't give you a concrete answer for that one unfortunately. $\endgroup$

Commented Jul 11, 2015 at 20:23

1 Answer 1

$\begingroup$

METHOD 1: EXPLOIT GREEN"S THEOREM

On the line integral from $(1,1)$ to $(4,1)$, $\vec F=\hat x(x+1)$. So, the integral over the three segments of interest can be computed as the integral over all four segments minus the integral over the segment from $(1,1)$ to $(4,1)$. And that integral is trivial. The integral over the four segments constitutes a closed curve and application of Green's Theorem is again trivial.

Let $C_4$ be the segment from $(1,1)$ to $(4,1)$. Then, we have

$$\begin \int_C \vec F \cdot d\vec r&=\oint_\vec F \cdot d\vec r-\int_\vec F \cdot d\vec r\\\\ &=\int_1^3\int_1^4\,(-1)\,dx\,dy-\int_1^4(x+1)dx\\\\ &=-6-\frac\\\\ &=-\frac \end$$

METHOD 2: DIRECT LINE INTEGRAL APPROACH

If we wish to compute the line integral over the original contour $C$ directly, we have

$$\int_C \vec F \cdot d\vec r=\int_1^3 (y-1)^3e^dy+\int_4^1(x+3)dx+\int_3^1(y-1)^3e^dy$$

The first and third integrals on the right-hand side cancel and we have

It might be interesting to note that this answer would not be impacted if we changed $F_y$ to any smooth function that is independent of $x$.